Matemática, perguntado por KaduPenna, 1 ano atrás

Me ajudem com essa questão por favor?

Anexos:

Soluções para a tarefa

Respondido por walkyrialago
2
-3+( -2)^{3}+(-2)^4:[(-2-3)^3:(-1-4)+(-3)^2.(-2-1)^3]

-3-8+16:[(-5)^3:(-5)+9.(-3)^3]

-11+16:[-125:(-5)+9 .(-27)]

-11+16:[+25-243]

-11+16:[-218]

-11-0,07339

-11,07339

Perguntas interessantes